Đến nội dung

vda2000 nội dung

Có 295 mục bởi vda2000 (Tìm giới hạn từ 05-06-2020)



Sắp theo                Sắp xếp  

#540072 Cho đa giác lồi 100 đỉnh

Đã gửi bởi vda2000 on 09-01-2015 - 11:27 trong Hình học

Đề bài:

Cho đa giác lồi 100 đỉnh. Chứng minh có thể chọn ra 3 đỉnh sao cho đường tròn đi qua 3 đỉnh đó chứa các đỉnh còn lại của đa giác.




#540075 Tuyển tập các bài hình thi vào chuyên THPT

Đã gửi bởi vda2000 on 09-01-2015 - 12:05 trong Hình học

Đề bài:

Cho đa giác lồi 100 đỉnh. Chứng minh có thể chọn ra 3 đỉnh sao cho đường tròn đi qua 3 đỉnh đó chứa các đỉnh còn lại của đa giác.




#540112 Hình học liên quan đến đa giác lồi, đường tròn.

Đã gửi bởi vda2000 on 09-01-2015 - 17:08 trong Hình học

Đề bài:

Cho đa giác lồi $100$ đỉnh. Chứng minh rằng có thể chọn ra $3$ đỉnh sao cho đường tròn đi qua $3$ đỉnh đó chứa các đỉnh còn lại của đa giác.




#541189 Đề thi HSG Toán 9 của Tp Bắc Giang, năm học 2014-2015

Đã gửi bởi vda2000 on 18-01-2015 - 16:32 trong Tài liệu - Đề thi

Đề thi thành phố Bắc Giang năm học 2014-2015.

 

Câu 1:

a/ Cho biểu thức H = $(\dfrac{a-b}{\sqrt{a^{2}-b^{2}}-a+b}+\dfrac{\sqrt{a-b}}{\sqrt{a+b}+\sqrt{a-b}}).\dfrac{a^{2}+3b^{2}}{\sqrt{a^{2}-b^{2}}}$

Biết $a-b=1$. Tìm GTNN của H.

b/ Cho ba số thực dương $a,b,c$ thỏa mãn: $a+b+c=\sqrt{a}+\sqrt{b}+\sqrt{c}=2$

Tính giá trị của: M=$\sum\dfrac{1+a}{\sqrt{a}+\sqrt{b}}$

 

Câu 2:

a/ Tìm $x,y$ nguyên thỏa mãn: $x^{2}(y^{2}-5)=y(y-x)$

b/ Giải phương trình: $4\sqrt{x^{3}+3x^{2}}+2\sqrt{2x-1}=4x^{2}+3x+3$

Câu 3:

a/ Tìm$a$ biết $a +\sqrt{24}$ và $\dfrac{1}{a}-\sqrt{24}$ là các số nguyên

b/ Cho $a,b,c$ dương thỏa mãn: $\dfrac{1}{a}+\dfrac{1}{b}+\dfrac{1}{c}=\dfrac{3}{2}$

Tìm GTLN của:

H =$\dfrac{1}{\sqrt{5a^{2}+2ab+2b^{2}}}+\dfrac{1}{\sqrt{5b^{2}+2bc+2c^{2}}}+\dfrac{1}{\sqrt{5c^{2}+2ca+2a^{2}}}$

Câu 4:

Cho tam giác $ABC$ vuông tại $A$ ($AB\geqslant AC$). Độ dài 3 cạnh lần lượt là $c,b,a$. Vẽ đường cao $AH$. Vẽ đường tròn $(O;R)$ nội tiếp tam giác $ABC$ và tiếp xúc các cạnh $AB,BC,CA$ lần lượt tại $M,N,E$. Gọi $I$ là trung điểm $AC$. $IO$ căt $AB$ tại $K$. $ME$ cắt $AH$ tại $G$. Chứng minh rằng

a/  $AO$=$\dfrac{\sqrt{2}}{2} (b+c-\sqrt{b^{2}+c^{2}})$

b/ Chứng minh SABC=$EB.EC$

c/ $AK=AG$

Câu 5: 

Cho $2014$ số không âm thỏa mãn: 

+) Tổng các số bằng $3$.

+) Tổng bình phương các số bằng $1$.

Chứng minh trong $2014$ số trên luôn tồn tại $3$ số có tổng lớn hơn hoặc bằng $1$.




#541266 Đề thi chọn HSG cấp thành phố Toán học 9 năm học 2014-2015

Đã gửi bởi vda2000 on 18-01-2015 - 22:52 trong Tài liệu - Đề thi

Phòng GD&ĐT                                                                         Đề thi học sinh giỏi cấp thành phố 

TP. Bắc Giang                                                                                     Năm học: 2014-2015

                                                                                                               Môn: Toán lớp 9

                                                                                                       Thời gian làm bài: 150 phút

Câu 1:

a/ Cho biểu thức H = $(\dfrac{a-b}{\sqrt{a^{2}-b^{2}}-a+b}+\dfrac{\sqrt{a-b}}{\sqrt{a+b}+\sqrt{a-b}}).\dfrac{a^{2}+3b^{2}}{\sqrt{a^{2}-b^{2}}}$ với $a>b>0$

Biết $a-b=1$. Tìm GTNN của H.

b/ Cho ba số thực dương $a,b,c$ thỏa mãn: $a+b+c=\sqrt{a}+\sqrt{b}+\sqrt{c}=2$

Tính giá trị của: M=$\sum\dfrac{1+a}{\sqrt{a}+\sqrt{b}}$

 

Câu 2:

a/ Tìm $x,y$ nguyên thỏa mãn: $x^{2}(y^{2}-5)=y(y-x)$

b/ Giải phương trình: $4\sqrt{x^{3}+3x^{2}}+2\sqrt{2x-1}=4x^{2}+3x+3$

 

Câu 3:

a/ Tìm $a$ biết $a +\sqrt{24}$ và $\dfrac{1}{a}-\sqrt{24}$ là các số nguyên

b/ Cho $a,b,c$ dương thỏa mãn: $\dfrac{1}{a}+\dfrac{1}{b}+\dfrac{1}{c}=\dfrac{3}{2}$

Tìm GTLN của:

H =$\dfrac{1}{\sqrt{5a^{2}+2ab+2b^{2}}}+\dfrac{1}{\sqrt{5b^{2}+2bc+2c^{2}}}+\dfrac{1}{\sqrt{5c^{2}+2ca+2a^{2}}}$

 

Câu 4:

Cho tam giác $ABC$ vuông tại $A$ ($AB<AC$). Độ dài 3 cạnh lần lượt là $c,b,a$. Vẽ đường cao $AH$. Vẽ đường tròn $(O;R)$ nội tiếp tam giác $ABC$ và tiếp xúc các cạnh $AB,BC,CA$ lần lượt tại $M,N,E$. Gọi $I$ là trung điểm $AC$. $IO$ căt $AB$ tại $K$. $ME$ cắt $AH$ tại $G$. Chứng minh rằng

a/  $AO$=$\dfrac{\sqrt{2}}{2} (b+c-\sqrt{b^{2}+c^{2}})$

b/ Chứng minh SABC=$EB.EC$

c/ $AK=AG$

 

Câu 5: 

Cho $2014$ số không âm thỏa mãn: 

+) Tổng các số bằng $3$.

+) Tổng bình phương các số bằng $1$.

Chứng minh trong $2014$ số trên luôn tồn tại $3$ số có tổng lớn hơn hoặc bằng $1$.




#541670 Chứng minh phương trình: $(a^2+b^2-1)x^2-2(ac+bd-1)x+c^2+d^2-1=0$ l...

Đã gửi bởi vda2000 on 24-01-2015 - 11:43 trong Số học

Bạn nhầm rồi.$\Delta =(a-c)^2+(b-d)^2-(ad-bc)^2$ chứ

 Anh ơi cho em hỏi: Bài này bọn anh chữa chưa và nếu chữa rồi anh có thể nói sơ qua cách làm được không ạ?
P/S: cảm ơn anh trước.




#541886 Đề thi học sinh giỏi casio khu vực phía bắc năm 2013-2014

Đã gửi bởi vda2000 on 25-01-2015 - 21:50 trong Tài liệu - Đề thi

Mọi người cho em hỏi thể thức thi QG là 5 câu tự luận hay 10 câu ghi đáp án như Thi TP, Thi tỉnh? /Đối với năm 2014-2015/




#541889 $(a^{2}+b^{2}-1)x^{2}-2(ac+bd-1)x+c^{...

Đã gửi bởi vda2000 on 25-01-2015 - 21:57 trong Phương trình, hệ phương trình và bất phương trình

Không liên quan nhưng cho mình hỏi bạn có phải Vương ĐÌnh Ân không ? 

=)) thằng bạn em chị ạ




#543636 Tìm a, b, c thỏa mãn $\dfrac{a}{b} +\dfrac...

Đã gửi bởi vda2000 on 10-02-2015 - 12:27 trong Đại số

Đề bài:

Tìm $a,b,c>0$ thỏa mãn: $a+b+c=1$ và $\dfrac{a}{b} +\dfrac{b}{c} + \dfrac{c}{a}=\dfrac{a+b}{b+c} + \dfrac{b+c}{a+b}+1$

 




#543656 Tìm a, b, c thỏa mãn $\dfrac{a}{b} +\dfrac...

Đã gửi bởi vda2000 on 10-02-2015 - 18:08 trong Đại số

Cảm ơn bạn nhiều !




#544066 Chứng minh: $\frac{9R}{a^2+b^2+c^2}\geq...

Đã gửi bởi vda2000 on 13-02-2015 - 23:47 trong Bất đẳng thức và cực trị

Đề bài:

Cho $\Delta ABC$. Độ dài 3 cạnh là $a,b,c$; $h_a;h_b;h_c$ là các đường cao tương ứng; $R,r$ lần lượt là bán kính đường tròn ngoại tiếp, nội tiếp $\Delta ABC$.

Chứng minh rằng: $\frac{9R}{a^2+b^2+c^2}\leq\sum\frac{1}{h_a+\sqrt{h_b h_c}}\leq\frac{1}{2r}$

 

Vế sau mình biết làm rồi.




#544154 Chứng minh: $ab^2+bc^2+ca^2\leq 2+abc$

Đã gửi bởi vda2000 on 14-02-2015 - 18:24 trong Bất đẳng thức và cực trị

Đề bài:

Cho $a,b,c$ là các số không âm thỏa mãn: $a^2+b^2+c^2=3$. Chứng minh rằng:

$ab^2+bc^2+ca^2\leq 2+abc$




#544273 Chứng minh rằng: $27|xyz$

Đã gửi bởi vda2000 on 15-02-2015 - 15:17 trong Số học

Đề bài:

 

Cho $x,y,z$ là các số nguyên thỏa mãn: $(x-y)(y-z)(z-x)=xyz$. Chứng minh rằng: $xyz$ chia hết cho $27$.

 

P/S: Cần gấp!




#544457 Chứng minh: $\frac{1}{AD}+\frac{1...

Đã gửi bởi vda2000 on 16-02-2015 - 12:03 trong Hình học

Bài toán:

 

Cho đường tròn $(O;R)$ và một điểm A nằm ngoài đường tròn sao cho $OA>2R$. Từ $A$ vẽ hai tiếp tuyến $AB,AC$ với đường tròn ($B,C$ là $2$ tiếp điểm). Vẽ dây $BD$ của đường tròn $(O;R)$ sao cho $BD//AC$. $E$ là giao điểm thứ hai của $AD$ với đường tròn $(O)$. Tia $BE$ cắt $AC$ tại $F$. 

 

1/ Chứng minh rằng: $FA=FC$.

2/ Kẻ đường kính $CK$ của đường tròn $(O;R)$. $M,N$ lần lượt là giao điểm của $FK$ với $CE$ và $ED$. Chứng minh rằng: $MF.KN=MN.KF$.

3/ Đường thẳng $AD$ cắt $BC$ tại $H$. Chứng minh:$\frac{1}{AD}+\frac{1}{HD}=\frac{2}{ED}.$

 

Mình còn câu 3, ai giúp hộ với!




#544567 Chứng minh: $\frac{1}{AD}+\frac{1...

Đã gửi bởi vda2000 on 16-02-2015 - 21:46 trong Hình học

:v what is tứ giác điều hòa? 




#544632 Chứng minh: $\frac{1}{MR}=\frac{1...

Đã gửi bởi vda2000 on 17-02-2015 - 11:33 trong Hình học

Cho đường tròn $(O;R)$, hai đường kính $AB,CD$ vuông góc nhau. $M$ là một điểm nằm trên đoạn thẳng $AB$ sao cho $\widehat {MCO} = 30 $ độ. Gọi $I$ là một điểm thay đổi trên đường kính $CD$. $MI$ cắt đường tròn tại $2$ điểm là $R,S$ ($MR<MS$).

 

Chứng minh:$\frac{1}{MR}=\frac{1}{MI}+\frac{1}{MS}$.




#544736 Chứng minh: $\frac{1}{MR}=\frac{1...

Đã gửi bởi vda2000 on 17-02-2015 - 23:26 trong Hình học

cảm ơn bạn!




#544829 Chứng minh rằng: $27|xyz$

Đã gửi bởi vda2000 on 18-02-2015 - 18:25 trong Số học

Tự đăng tự giải vậy :D !
 
Ta có: $(x-y)(y-z)(z-x)=xyz$
 
Nếu trong 3 số $x,y,z$ không có số nào chia hết cho $3$ thì theo nguyên lí Đi-rích-lê, tồn tại ít nhất $2$ số có cùng số dư nên hiệu của chúng chia hệt cho $3$.
$=> 3|(x-y)(y-z)(z-x)$
$=> 3|xyz$ (vì $(x-y)(y-z)(z-x)=xyz$)
$=>$ Vô lí
$=>$ Loại.
 
Do đó trong $3$ số $x,y,z$ tồn tại ít nhất một số chia hết cho $3$ (giả sử $3|x$ thì $3|xyz$) .
Xét các trường hợp sau:
 
+) $y,z$ chia $3$ dư $1;2$
=> $x-y;y-z;z-x$ đều không chia hết cho $3$ 
$=> (x-y)(y-z)(z-x)$ không chia hết cho $3$
$=>$ $xyz$ không chia hết cho $3$ (Vô lí)
 
+) $y;z$ không chia hết cho 3 nhưng cùng số dư.
Dễ dàng chứng minh: $(x-y)^3+(y-z)^3+(z-x)^3=3(x-y)(y-z)(z-x)$
$=>3|(x-y)^3+(y-z)^3+(z-x)^3$  
Vì $3|x$ và $3|y-z$ và $y,z$ không chia hết cho $3$ nên $(x-y)^3+(y-z)^3+(z-x)^3$ không chia hết cho $3$ 
( vì nó sẽ đồng dư với $1^3+0^3+1^3=2$ hoặc $2^3+0^3+2^3=16$ nên chia dư $2$ hoặc $1$).
$=>$ Vô lí.
$=>$ Loại.
 
Do đó, $3|x,y,z$
           $=>$ $27|xyz$ 
           $=>$ $Q.E.D$




#544830 Chứng minh rằng: $27|xyz$

Đã gửi bởi vda2000 on 18-02-2015 - 18:56 trong Số học

Tự đăng tự giải vậy :D !
 
Ta có: $(x-y)(y-z)(z-x)=xyz$
 
Nếu trong 3 số $x,y,z$ không có số nào chia hết cho $3$ thì theo nguyên lí Đi-rích-lê, tồn tại ít nhất $2$ số có cùng số dư nên hiệu của chúng chia hệt cho $3$.
$=> 3|(x-y)(y-z)(z-x)$
$=> 3|xyz$ (vì $(x-y)(y-z)(z-x)=xyz$)
$=>$ Vô lí
$=>$ Loại.
 
Do đó trong $3$ số $x,y,z$ tồn tại ít nhất một số chia hết cho $3$ (giả sử $3|x$ thì $3|xyz$) .
Xét các trường hợp sau:
 
+) $y,z$ chia $3$ dư $1;2$
=> $x-y;y-z;z-x$ đều không chia hết cho $3$ 
$=> (x-y)(y-z)(z-x)$ không chia hết cho $3$
$=>$ $xyz$ không chia hết cho $3$ (Vô lí)
 
+) $y;z$ không chia hết cho 3 nhưng cùng số dư.
Dễ dàng chứng minh: $(x-y)^3+(y-z)^3+(z-x)^3=3(x-y)(y-z)(z-x)$
$=>3|(x-y)^3+(y-z)^3+(z-x)^3$  
Vì $3|x$ và $3|y-z$ và $y,z$ không chia hết cho $3$ nên $(x-y)^3+(y-z)^3+(z-x)^3$ không chia hết cho $3$ 
( vì nó sẽ đồng dư với $1^3+0^3+1^3=2$ hoặc $2^3+0^3+2^3=16$ nên chia dư $2$ hoặc $1$).
$=>$ Vô lí.
$=>$ Loại.
 
Do đó, $3|x,y,z$
           $=>$ $27|xyz$ 
           $=>$ $Q.E.D$

Mình nhầm rồi ở chỗ gạch đỏ, haizzzzzzzzzzzz!




#544905 Giải phương trình nghiệm nguyên $x^3+x^2y+xy^2+y^3=8(x^2+xy+y^2+1)$

Đã gửi bởi vda2000 on 19-02-2015 - 20:31 trong Số học

Giải phương trình nghiệm nguyên $x^3+x^2y+xy^2+y^3=8(x^2+xy+y^2+1) (1)$

 

Ta có: $(1)  <=> (x^2+y^2)(x+y)=4(x^2+2xy+y^2)+4(x^2+y^2)+8$

                  $<=> (x^2+y^2)(x+y)=4(x+y)^2+4(x^2+y^2)+8 (2)$

Đặt $x^2+y^2=b$ và $x+y=a$. 

Ta có: $(2)  <=> ab=4a^2+4b+8$

                  $<=> (4a^2-64)+(4b-ab)= -8-64$

                  $<=> 4(a-4)(a+4) - b(a-4)= -72$

                  $<=> (a-4)(4a-b+16)=-72$

Đến đây giải phương trình tích nghiệm nguyên $a,b$ rồi sử dụng $Viète$ tìm $x,y$ rồi đối chiếu.




#544930 $$\dfrac{1}{b^2+c^2+2}+\dfrac{1...

Đã gửi bởi vda2000 on 19-02-2015 - 22:57 trong Bất đẳng thức và cực trị

 Nhầm :v 




#545026 Giải bất phương trình : $x\left (x^{8} +x^{2}+...

Đã gửi bởi vda2000 on 20-02-2015 - 18:01 trong Phương trình, hệ phương trình và bất phương trình

Giải bất phương trình : 

$x\left (x^{8} +x^{2}+16\right )> 6\left ( 4-x^{2} \right )  (1)$

$(1)$ <=> $x^9+x^3+16x-24+6x^2>0$

         <=> $(x^9-1) + (x^3+6x^2+16x-23) >0$

         <=> $(x-1)(x^8+x^7+x^6+...+1)+(x-1)(x^2+8x+23)>0$

         <=> $(x-1)(x^8+x^7+x^6+x^5+x^4+x^3+2x^2+8x+24)>0$

         <=> $(x-1).A>0$ $(2)$

Ta có: $16A= 16x^8+16x^7+16x^6+16x^5+16x^4+16x^3+32x^2+128x+192$

               $=(16x^8+16x^7+4x^6) + 12(x^6+\frac{4}{3}x^5+\frac{4}{9}x^4)+\frac{32}{3}(x^4+\frac{3}{2}x^3+\frac{9}{16}x^2)+26(x^2+\frac{64}{13}x+\frac{1024}{169}) + \frac{448}{13}$

               $=$ $(4x^4+4x^3)^2+12(x^3+\frac{2}{3}x^2)+\frac{32}{3}(x^2+\frac{3}{4}x)^2+26(x+\frac{32}{13})^2 + \frac{448}{13} >0$

=> $16A>0$

=> $A>0$

 

Khi đó: $(2)$ <=> $x-1>0$

                      <=> $x>1$

 

Vậy bất phương trình $(1)$ có nghiệm $x\in\mathbb{R}|x>1$




#545084 Với giá trị nào của x thì A<$\frac{1}{2}$

Đã gửi bởi vda2000 on 20-02-2015 - 23:20 trong Đại số

Cho biểu thức A=$(\frac{x+1}{x-1}+\frac{x-1}{x+1}+\frac{x^{2}-4x-1}{x^2-1})$

Với giá trị nào của x thì $A$<$\frac{1}{2}$

Rút gọn: $A=\frac{3x-1}{x+1}$ với $x\neq 1$ và $x\neq -1$.

 

Để $A<\frac{1}{2}$

<=> $\frac{3x-1}{x+1}-\frac{1}{2} <0$

<=> $\frac{5x-3}{2(x+1)}<0$

<=> $(5x-3)(x+1)<0$

<=> $-1<x<\frac{3}{5}$ (Thỏa mãn điều kiện của $x$)

 

Vậy bất phương trình có nghiệm: $x\in\mathbb{R}|-1<x<\frac{3}{5}$




#545098 Cho $a+2b+3c=100$. Tìm GTLN của $abc$

Đã gửi bởi vda2000 on 21-02-2015 - 09:15 trong Bất đẳng thức và cực trị

1/ Cho $a+2b+3c=100$. Tìm GTLN của $abc$

 

2/ Cho a,b,c là các số nguyên dương. Chứng minh:

 

$\sum \frac{(b+c-a)^2}{(b+c)^2+a^2}\geq \frac{3}{5}$

1/ Max:

Áp dụng bất đẳng thức $Cauchy$ với $3$ số dương, ta có:

$a+2b+3c\geq 3\sqrt[3]{6abc}$

<=> $100\geq 3\sqrt[3]{6abc}$

<=> $abc\leq \frac{500000}{81}$

 

Dấu $"="$ xảy ra <=> $a=\frac{100}{3}; b=\frac{50}{3}; c=\frac{100}{9}$

Kết luận: ....




#545103 Cho $a+2b+3c=100$. Tìm GTLN của $abc$

Đã gửi bởi vda2000 on 21-02-2015 - 10:42 trong Bất đẳng thức và cực trị

2/ Cho a,b,c là các số nguyên dương. Chứng minh:

 

$\sum \frac{(b+c-a)^2}{(b+c)^2+a^2}\geq \frac{3}{5}$

 

Ta có: $A=\sum\frac{(b+c-a)^2}{(b+c)^2+a^2}$

    <=> $A=\sum\frac{(b+c)^2+a^2-2a(b+c)}{(b+c)^2+a^2}$

    <=> $A=\sum 1-\frac{2a(b+c)}{(b+c)^2+a^2}$

    <=> $A=3 -\sum\frac{2a(b+c)}{(b+c)^2+a^2}$

 

Áp dụng bắt đẳng thức $AM-GM$, ta có:

$a^2+\frac{(b+c)^2}{4}\geq a(b+c)$

<=> $a^2+(b+c)^2\geq a(b+c)+\frac{3}{4}(b+c)^2$

 

Do đó: $A\geq 3-\sum\frac{2a(b+c)}{a(b+c)+\frac{3}{4}.(b+c)^2}$

<=> $A\geq 3-\sum 2-\frac{\frac{3}{2}.(b+c)^2}{a(b+c)+\frac{3}{4}.(b+c)^2}$

<=> $A\geq\sum\frac{\frac{3}{2}(b+c)^2}{a(b+c)+\frac{3}{4}.(b+c)^2}-3$

<=> $A\geq\frac{3}{2}.B-3  (1)$

 

Ta có: $B=\sum\frac{(b+c)^2}{(b+c)(a+\frac{3}{4}(b+c))}$

     <=>$B=\sum\frac{b+c}{a+\frac{3}{4}(b+c)}$

     <=>$\frac{1}{4}B+3=\sum{\frac{\frac{1}{4}(b+c)}{a+\frac{3}{4}(b+c)}+1}$

     <=>$\frac{1}{4}B+3=(a+b+c).\sum\frac{1}{a+\frac{3}{4}.(b+c)}\geq (a+b+c).\frac{9}{\sum{a+\frac{3}{4}(b+c)}}$

Lại có: $VP=(a+b+c).\frac{9}{\frac{5}{2}(a+b+c)}=\frac{18}{5}$.

 

Do đó, $\frac{1}{4}B+3\geq\frac{18}{5}$

<=> $B\geq(\frac{18}{5}-3).4=\frac{12}{5}  (2)$

 

Từ $(1),(2)$, ta có: $A\geq \frac{3}{2}.\frac{12}{5}-3=\frac{3}{5}$

=> $Q.E.D$

 

Vậy $\sum \frac{(b+c-a)^2}{(b+c)^2+a^2}\geq \frac{3}{5}$

Dấu $"="$ xảy ra <=> $a=b=c\in\mathbb{Z_+}$